LSAT and Law School Admissions Forum

Get expert LSAT preparation and law school admissions advice from PowerScore Test Preparation.

 Administrator
PowerScore Staff
  • PowerScore Staff
  • Posts: 8917
  • Joined: Feb 02, 2011
|
#40240
Complete Question Explanation
(The complete setup for this game can be found here: lsat/viewtopic.php?t=8562)

The correct answer choice is (A)

To attack this question, use proper List question technique: take one rule and apply it to all answer choices, then take another rule, and apply it to the remaining answer choices, and so on, until only one answer choice remains. However, do not just take the rules in the order given. Instead, choose the rules to apply in order of the ease of seeing them visually inside each answer choice. In this game, the rules should be applied in this order: third, second, first, fifth, fourth.

Answer choice (A): This is the correct answer choice.

Answer choice (B): This answer choice violates the first rule and is therefore incorrect.

Answer choice (C): This answer choice violates the fifth rule and is therefore incorrect.

Answer choice (D): This answer choice violates the fourth rule and is therefore incorrect.

Answer choice (E): This answer choice violates the third rule and is therefore incorrect.

As usual, each of the four incorrect answer choices violates a different rule.
 theamazingrace
  • Posts: 59
  • Joined: Oct 17, 2020
|
#81321
Why is C wrong? it fits with all the rules. I know in questions 21 and 22 P has to be in the third bouquet but why isn't R and S enough in this case?

Thanks
User avatar
 KelseyWoods
PowerScore Staff
  • PowerScore Staff
  • Posts: 1079
  • Joined: Jun 26, 2013
|
#81421
Hi theamazingrace!

Answer choice (C) violates the 5th rule: "If a bouquet has tulips, that bouquet must also have peonies." In (C), bouquet 2 has roses, snapdragons, and tulips, but no peonies. According to rule #5, if a bouquet has tulips, it must have peonies. Since there are tulips but no peonies in bouquet 2, answer choice (C) violates that rule and so is incorrect.

Hope this helps!

Best,
Kelsey

Get the most out of your LSAT Prep Plus subscription.

Analyze and track your performance with our Testing and Analytics Package.